Let $ X sim operatorname{Exp}(lambda =1), Ysim U(1,2) $ be independent continuous variables. What is...












0












$begingroup$


I'm struggling to understand how to start the following:




Let $ X sim operatorname{Exp}(lambda =1), Ysim U(1,2) $ be independent
continuous variables. What is $E(frac{x}{y})$?




Thanks :)










share|cite|improve this question











$endgroup$

















    0












    $begingroup$


    I'm struggling to understand how to start the following:




    Let $ X sim operatorname{Exp}(lambda =1), Ysim U(1,2) $ be independent
    continuous variables. What is $E(frac{x}{y})$?




    Thanks :)










    share|cite|improve this question











    $endgroup$















      0












      0








      0





      $begingroup$


      I'm struggling to understand how to start the following:




      Let $ X sim operatorname{Exp}(lambda =1), Ysim U(1,2) $ be independent
      continuous variables. What is $E(frac{x}{y})$?




      Thanks :)










      share|cite|improve this question











      $endgroup$




      I'm struggling to understand how to start the following:




      Let $ X sim operatorname{Exp}(lambda =1), Ysim U(1,2) $ be independent
      continuous variables. What is $E(frac{x}{y})$?




      Thanks :)







      probability expected-value






      share|cite|improve this question















      share|cite|improve this question













      share|cite|improve this question




      share|cite|improve this question








      edited Jan 15 at 10:07









      Bernard

      123k741117




      123k741117










      asked Jan 15 at 10:02









      superuser123superuser123

      48628




      48628






















          2 Answers
          2






          active

          oldest

          votes


















          1












          $begingroup$

          $E(frac X Y |Y)=frac 1 Y EX=frac 1 Y$. Taking expectation we get $Efrac X Y=Efrac 1 Y=int_1^{2} frac 1 y , dy=ln, 2$.






          share|cite|improve this answer









          $endgroup$













          • $begingroup$
            Note that this only holds because $Y$ is never zero and thus the division expression is well-defined.
            $endgroup$
            – Thomas Lang
            Jan 15 at 10:08






          • 1




            $begingroup$
            Thanks, but why is the condition taking place? ($E(frac{X}{Y} | Y) $?
            $endgroup$
            – superuser123
            Jan 15 at 10:19










          • $begingroup$
            @superuser123 You can also do this using a double integral but using conditional expectation makes it easier.
            $endgroup$
            – Kavi Rama Murthy
            Jan 15 at 10:20










          • $begingroup$
            @KaviRamaMurthy how could it be done by a double integral? thanks!
            $endgroup$
            – superuser123
            Jan 15 at 10:22






          • 1




            $begingroup$
            @superuser123 Calculate (by integrating w.r.t $x$ and then w.r.t. $y$ or the other way around) $int_0^{infty} int_1^{2} frac x y e^{-x} ,dy, dx$
            $endgroup$
            – Kavi Rama Murthy
            Jan 15 at 10:25





















          0












          $begingroup$

          Guide:



          If $X$ and $Y$ are independent then so are $f(X)$ and $g(Y)$ where $f$ and $g$ are measurable functions.



          Consequence for expectation (if it exists):$$mathbb Ef(X)g(Y)=mathbb Ef(X)mathbb Eg(Y)$$



          This can be applied to find: $$mathbb E[XY^{-1}]=mathbb EXmathbb EY^{-1}$$
          It only remains now to find $mathbb EX$ and $mathbb EY^{-1}$.






          share|cite|improve this answer











          $endgroup$














            Your Answer





            StackExchange.ifUsing("editor", function () {
            return StackExchange.using("mathjaxEditing", function () {
            StackExchange.MarkdownEditor.creationCallbacks.add(function (editor, postfix) {
            StackExchange.mathjaxEditing.prepareWmdForMathJax(editor, postfix, [["$", "$"], ["\\(","\\)"]]);
            });
            });
            }, "mathjax-editing");

            StackExchange.ready(function() {
            var channelOptions = {
            tags: "".split(" "),
            id: "69"
            };
            initTagRenderer("".split(" "), "".split(" "), channelOptions);

            StackExchange.using("externalEditor", function() {
            // Have to fire editor after snippets, if snippets enabled
            if (StackExchange.settings.snippets.snippetsEnabled) {
            StackExchange.using("snippets", function() {
            createEditor();
            });
            }
            else {
            createEditor();
            }
            });

            function createEditor() {
            StackExchange.prepareEditor({
            heartbeatType: 'answer',
            autoActivateHeartbeat: false,
            convertImagesToLinks: true,
            noModals: true,
            showLowRepImageUploadWarning: true,
            reputationToPostImages: 10,
            bindNavPrevention: true,
            postfix: "",
            imageUploader: {
            brandingHtml: "Powered by u003ca class="icon-imgur-white" href="https://imgur.com/"u003eu003c/au003e",
            contentPolicyHtml: "User contributions licensed under u003ca href="https://creativecommons.org/licenses/by-sa/3.0/"u003ecc by-sa 3.0 with attribution requiredu003c/au003e u003ca href="https://stackoverflow.com/legal/content-policy"u003e(content policy)u003c/au003e",
            allowUrls: true
            },
            noCode: true, onDemand: true,
            discardSelector: ".discard-answer"
            ,immediatelyShowMarkdownHelp:true
            });


            }
            });














            draft saved

            draft discarded


















            StackExchange.ready(
            function () {
            StackExchange.openid.initPostLogin('.new-post-login', 'https%3a%2f%2fmath.stackexchange.com%2fquestions%2f3074263%2flet-x-sim-operatornameexp-lambda-1-y-sim-u1-2-be-independent-conti%23new-answer', 'question_page');
            }
            );

            Post as a guest















            Required, but never shown

























            2 Answers
            2






            active

            oldest

            votes








            2 Answers
            2






            active

            oldest

            votes









            active

            oldest

            votes






            active

            oldest

            votes









            1












            $begingroup$

            $E(frac X Y |Y)=frac 1 Y EX=frac 1 Y$. Taking expectation we get $Efrac X Y=Efrac 1 Y=int_1^{2} frac 1 y , dy=ln, 2$.






            share|cite|improve this answer









            $endgroup$













            • $begingroup$
              Note that this only holds because $Y$ is never zero and thus the division expression is well-defined.
              $endgroup$
              – Thomas Lang
              Jan 15 at 10:08






            • 1




              $begingroup$
              Thanks, but why is the condition taking place? ($E(frac{X}{Y} | Y) $?
              $endgroup$
              – superuser123
              Jan 15 at 10:19










            • $begingroup$
              @superuser123 You can also do this using a double integral but using conditional expectation makes it easier.
              $endgroup$
              – Kavi Rama Murthy
              Jan 15 at 10:20










            • $begingroup$
              @KaviRamaMurthy how could it be done by a double integral? thanks!
              $endgroup$
              – superuser123
              Jan 15 at 10:22






            • 1




              $begingroup$
              @superuser123 Calculate (by integrating w.r.t $x$ and then w.r.t. $y$ or the other way around) $int_0^{infty} int_1^{2} frac x y e^{-x} ,dy, dx$
              $endgroup$
              – Kavi Rama Murthy
              Jan 15 at 10:25


















            1












            $begingroup$

            $E(frac X Y |Y)=frac 1 Y EX=frac 1 Y$. Taking expectation we get $Efrac X Y=Efrac 1 Y=int_1^{2} frac 1 y , dy=ln, 2$.






            share|cite|improve this answer









            $endgroup$













            • $begingroup$
              Note that this only holds because $Y$ is never zero and thus the division expression is well-defined.
              $endgroup$
              – Thomas Lang
              Jan 15 at 10:08






            • 1




              $begingroup$
              Thanks, but why is the condition taking place? ($E(frac{X}{Y} | Y) $?
              $endgroup$
              – superuser123
              Jan 15 at 10:19










            • $begingroup$
              @superuser123 You can also do this using a double integral but using conditional expectation makes it easier.
              $endgroup$
              – Kavi Rama Murthy
              Jan 15 at 10:20










            • $begingroup$
              @KaviRamaMurthy how could it be done by a double integral? thanks!
              $endgroup$
              – superuser123
              Jan 15 at 10:22






            • 1




              $begingroup$
              @superuser123 Calculate (by integrating w.r.t $x$ and then w.r.t. $y$ or the other way around) $int_0^{infty} int_1^{2} frac x y e^{-x} ,dy, dx$
              $endgroup$
              – Kavi Rama Murthy
              Jan 15 at 10:25
















            1












            1








            1





            $begingroup$

            $E(frac X Y |Y)=frac 1 Y EX=frac 1 Y$. Taking expectation we get $Efrac X Y=Efrac 1 Y=int_1^{2} frac 1 y , dy=ln, 2$.






            share|cite|improve this answer









            $endgroup$



            $E(frac X Y |Y)=frac 1 Y EX=frac 1 Y$. Taking expectation we get $Efrac X Y=Efrac 1 Y=int_1^{2} frac 1 y , dy=ln, 2$.







            share|cite|improve this answer












            share|cite|improve this answer



            share|cite|improve this answer










            answered Jan 15 at 10:07









            Kavi Rama MurthyKavi Rama Murthy

            70.8k53170




            70.8k53170












            • $begingroup$
              Note that this only holds because $Y$ is never zero and thus the division expression is well-defined.
              $endgroup$
              – Thomas Lang
              Jan 15 at 10:08






            • 1




              $begingroup$
              Thanks, but why is the condition taking place? ($E(frac{X}{Y} | Y) $?
              $endgroup$
              – superuser123
              Jan 15 at 10:19










            • $begingroup$
              @superuser123 You can also do this using a double integral but using conditional expectation makes it easier.
              $endgroup$
              – Kavi Rama Murthy
              Jan 15 at 10:20










            • $begingroup$
              @KaviRamaMurthy how could it be done by a double integral? thanks!
              $endgroup$
              – superuser123
              Jan 15 at 10:22






            • 1




              $begingroup$
              @superuser123 Calculate (by integrating w.r.t $x$ and then w.r.t. $y$ or the other way around) $int_0^{infty} int_1^{2} frac x y e^{-x} ,dy, dx$
              $endgroup$
              – Kavi Rama Murthy
              Jan 15 at 10:25




















            • $begingroup$
              Note that this only holds because $Y$ is never zero and thus the division expression is well-defined.
              $endgroup$
              – Thomas Lang
              Jan 15 at 10:08






            • 1




              $begingroup$
              Thanks, but why is the condition taking place? ($E(frac{X}{Y} | Y) $?
              $endgroup$
              – superuser123
              Jan 15 at 10:19










            • $begingroup$
              @superuser123 You can also do this using a double integral but using conditional expectation makes it easier.
              $endgroup$
              – Kavi Rama Murthy
              Jan 15 at 10:20










            • $begingroup$
              @KaviRamaMurthy how could it be done by a double integral? thanks!
              $endgroup$
              – superuser123
              Jan 15 at 10:22






            • 1




              $begingroup$
              @superuser123 Calculate (by integrating w.r.t $x$ and then w.r.t. $y$ or the other way around) $int_0^{infty} int_1^{2} frac x y e^{-x} ,dy, dx$
              $endgroup$
              – Kavi Rama Murthy
              Jan 15 at 10:25


















            $begingroup$
            Note that this only holds because $Y$ is never zero and thus the division expression is well-defined.
            $endgroup$
            – Thomas Lang
            Jan 15 at 10:08




            $begingroup$
            Note that this only holds because $Y$ is never zero and thus the division expression is well-defined.
            $endgroup$
            – Thomas Lang
            Jan 15 at 10:08




            1




            1




            $begingroup$
            Thanks, but why is the condition taking place? ($E(frac{X}{Y} | Y) $?
            $endgroup$
            – superuser123
            Jan 15 at 10:19




            $begingroup$
            Thanks, but why is the condition taking place? ($E(frac{X}{Y} | Y) $?
            $endgroup$
            – superuser123
            Jan 15 at 10:19












            $begingroup$
            @superuser123 You can also do this using a double integral but using conditional expectation makes it easier.
            $endgroup$
            – Kavi Rama Murthy
            Jan 15 at 10:20




            $begingroup$
            @superuser123 You can also do this using a double integral but using conditional expectation makes it easier.
            $endgroup$
            – Kavi Rama Murthy
            Jan 15 at 10:20












            $begingroup$
            @KaviRamaMurthy how could it be done by a double integral? thanks!
            $endgroup$
            – superuser123
            Jan 15 at 10:22




            $begingroup$
            @KaviRamaMurthy how could it be done by a double integral? thanks!
            $endgroup$
            – superuser123
            Jan 15 at 10:22




            1




            1




            $begingroup$
            @superuser123 Calculate (by integrating w.r.t $x$ and then w.r.t. $y$ or the other way around) $int_0^{infty} int_1^{2} frac x y e^{-x} ,dy, dx$
            $endgroup$
            – Kavi Rama Murthy
            Jan 15 at 10:25






            $begingroup$
            @superuser123 Calculate (by integrating w.r.t $x$ and then w.r.t. $y$ or the other way around) $int_0^{infty} int_1^{2} frac x y e^{-x} ,dy, dx$
            $endgroup$
            – Kavi Rama Murthy
            Jan 15 at 10:25













            0












            $begingroup$

            Guide:



            If $X$ and $Y$ are independent then so are $f(X)$ and $g(Y)$ where $f$ and $g$ are measurable functions.



            Consequence for expectation (if it exists):$$mathbb Ef(X)g(Y)=mathbb Ef(X)mathbb Eg(Y)$$



            This can be applied to find: $$mathbb E[XY^{-1}]=mathbb EXmathbb EY^{-1}$$
            It only remains now to find $mathbb EX$ and $mathbb EY^{-1}$.






            share|cite|improve this answer











            $endgroup$


















              0












              $begingroup$

              Guide:



              If $X$ and $Y$ are independent then so are $f(X)$ and $g(Y)$ where $f$ and $g$ are measurable functions.



              Consequence for expectation (if it exists):$$mathbb Ef(X)g(Y)=mathbb Ef(X)mathbb Eg(Y)$$



              This can be applied to find: $$mathbb E[XY^{-1}]=mathbb EXmathbb EY^{-1}$$
              It only remains now to find $mathbb EX$ and $mathbb EY^{-1}$.






              share|cite|improve this answer











              $endgroup$
















                0












                0








                0





                $begingroup$

                Guide:



                If $X$ and $Y$ are independent then so are $f(X)$ and $g(Y)$ where $f$ and $g$ are measurable functions.



                Consequence for expectation (if it exists):$$mathbb Ef(X)g(Y)=mathbb Ef(X)mathbb Eg(Y)$$



                This can be applied to find: $$mathbb E[XY^{-1}]=mathbb EXmathbb EY^{-1}$$
                It only remains now to find $mathbb EX$ and $mathbb EY^{-1}$.






                share|cite|improve this answer











                $endgroup$



                Guide:



                If $X$ and $Y$ are independent then so are $f(X)$ and $g(Y)$ where $f$ and $g$ are measurable functions.



                Consequence for expectation (if it exists):$$mathbb Ef(X)g(Y)=mathbb Ef(X)mathbb Eg(Y)$$



                This can be applied to find: $$mathbb E[XY^{-1}]=mathbb EXmathbb EY^{-1}$$
                It only remains now to find $mathbb EX$ and $mathbb EY^{-1}$.







                share|cite|improve this answer














                share|cite|improve this answer



                share|cite|improve this answer








                edited Jan 15 at 10:29

























                answered Jan 15 at 10:22









                drhabdrhab

                104k545136




                104k545136






























                    draft saved

                    draft discarded




















































                    Thanks for contributing an answer to Mathematics Stack Exchange!


                    • Please be sure to answer the question. Provide details and share your research!

                    But avoid



                    • Asking for help, clarification, or responding to other answers.

                    • Making statements based on opinion; back them up with references or personal experience.


                    Use MathJax to format equations. MathJax reference.


                    To learn more, see our tips on writing great answers.




                    draft saved


                    draft discarded














                    StackExchange.ready(
                    function () {
                    StackExchange.openid.initPostLogin('.new-post-login', 'https%3a%2f%2fmath.stackexchange.com%2fquestions%2f3074263%2flet-x-sim-operatornameexp-lambda-1-y-sim-u1-2-be-independent-conti%23new-answer', 'question_page');
                    }
                    );

                    Post as a guest















                    Required, but never shown





















































                    Required, but never shown














                    Required, but never shown












                    Required, but never shown







                    Required, but never shown

































                    Required, but never shown














                    Required, but never shown












                    Required, but never shown







                    Required, but never shown







                    Popular posts from this blog

                    Human spaceflight

                    Can not write log (Is /dev/pts mounted?) - openpty in Ubuntu-on-Windows?

                    File:DeusFollowingSea.jpg